Letter Lines

The letters A-I represent the digits 1-9 in some order. The letters are placed in a line from left to right. Determine which digit is represented by each letter and the order of the letters from left to right, given the following clues. ("A _ 6" means "A is two positions to the left of 6." "_ _ 8" means "There are at least two positions to the left of 8.")
Puzzle 1
A _ _ _ 5
4 _ _ B
C
D _ _ _ 2
6 _ _ _ E
7 _ _ _ _ F
5 _ G _
_ _ 8 _ H
1 _ I _ 9
Puzzle 2
A _ 6
B _ _ _ 4
C _ _ _ _ 7
1 _ D _ 7
1 _ _ _ _ E
9 _ F
G _ _ 8
2 _ _ H
5 _ _ _ I

Source: Original. [Has anyone seen this sort of puzzle before? It seems new to me.]


Solutions were received from Denis Borris, Philippe Fondanaiche. I didn't get a logical approach, but here are the correct answers:

Puzzle 1: 
 C A D B H I F G E
 4 7 8 1 6 5 2 9 3
or
 4 6 8 7 1 5 2 3 9
 C A D B H E I G F

Puzzle 2:
 G C F B D I A E H
 9 5 1 8 3 2 7 4 6

Update 4/30/01: Hareendra Yalamanchili sent logical solutions:
  1. Logic for puzzle 1
    
    From A---5 and 5-G- we have A---5-G-
    G is at spot 7 or 8. 
    The first 2 spots must be A,C,or D and the last 2 are 2,3,or 9
    
    1-I-9 has 5 possibilities:
    1.  |  1 | A  | I  |    |   9 |  5 |    |  G |    |
      F has 2 possibilities:
     1a.|  1 | A7 | I  |    |   9 |  5 | F  |  G |    |
       leaves no spot for E
     1b.|  1 | A  | I  |  7 |   9 |  5 |    |  G |  F |
       forces H
        |  1 | A  | I8 |  7 |  H9 |  5 |    |  G |  F |
       forces E
        |  1 | A6 | I8 |  7 |  H9 | E5 |    |  G |  F |
       leaves no spot for B.
    
    2.  | A  |  1 |    | I  |   5 |  9 |    | G  |    |
      2 and 3 must be at the last 2 spots, forcing
        | A  |  1 |    | I  |  D5 |  9 |    | G3 |  2 |
      forces E
        | A  |  1 |  6 | I  |  D5 |  9 | E  | G3 |  2 |
      leaves no space for B
    
    Since I is not at spots 3 or 4 this means that the last 5 spots are
      E,F,G,H, and I
    This forces B to spot 4.
        |  4 |    |    | B  |    |    |    |    |    |
    D cannot be at spots 4 or 5, so 2 is not at spot 8 or 9, making the last
    2 spots 3 and 9, thus eliminating case 3.
     
    4.  |  4 |    |    | B1 |    | I  |    |  9 |  3 |
        F&G can only be in the 7th and 8 spots, forcing E and then H
        |  4 |    |  8 | B1 | H6 | I  |    |  9 | E3 |
        forces A---5-G- 
        |  4 | A  |  8 | B1 | H6 | I5 |    | G9 | E3 |
      giving a solution
    | C4 | A7 | D8 | B1 | H6 | I5 | F2 | G9 | E3 |
    
    5.  |  4 |    |    | B  |  1 |    | I  |  3 |  9 |
      forces A---5--G
        |  4 | A  |    | B  |  1 |  5 | I  | G3 |  9 |
      forces D,E,F and gives the solution
    | C4 | A6 | D8 | B7 | H1 | E5 | I2 | G3 | F9 |
    
  2. Puzzle 2
                                          A _ 6
                                          B _ _ _ 4
                                          C _ _ _ _ 7
                                          1 _ D _ 7
                                          1 _ _ _ _ E
                                          9 _ F
                                          G _ _ 8
                                          2 _ _ H
                                          5 _ _ _ I
    
    Note the the first two must be A,B,C, or G
    The C-E hints give us
      
    |  C | 1  |   | D  |   | 7  |  E   |
    Which can fit in 3 ways
    1.|    |    | C  | 1  |    | D  |    | 7  | E  |
    Then G| | |8 can fit in 2 places.
    1a. |    | G  | C  | 1  |  8 | D  |    | 7  | E  |
      This forces B into 
       |    | G  | C  | 1  | B8 | D  |    | 7  | E4 |
      and A must be on the left
       | A  | G  | C6 | 1  | B8 | D  |    | 7  | E4 |
      leaving no place for I.
    
    1b. |    |    | C  | 1G |    | D  |  8 |  7 | E  |
      Then A&B must be on the left, 
        | A  | B  | C6 | 1G |    | D4 |  8 |  7 | E  |
      forces F
        | A  | B  | C6 | 1G |  9 | D4 | F8 |  7 | E  |
      leaves no space for H&I
    
    2.   | C  | 1  |    | D  |    | 7  | E  |    |    |
      b---4 fits in 2 ways, and the 2nd spot is either A or G
     2a. | C  | G1 | B  | D  |  8 | 7  | E4 |    |    |
      forces A&H into the same square
     2b. | C  | A1 | B  | D6 |    |  7 | E4 |    |    |
      forces F and forces the last square to be I
         | C  | A1 | B9 | D6 | F5 |  7 | E4 |    | I  |
      forces G&H into the same square
     2c. | C  | A1 |    | D6 | B  |  7 | E  |    |  4 |
      leaves no square for G
     2d. | C  | G1 |    | D  | B8 |  7 | E  |    |  4 |
      The 3rd and last squares cannot be A, H, or I forcing
        F into both squares
    3.   |    | C  | 1  |    | D  |    | 7  | E  |    |
      b---4 fits in 2 ways
     3a. | B  | C  | 1  |    | D4 |    | 7  | E  |    |
      forces the last square to be H
         | B  | C  | 1  |    | D4 |  2 | 7  | E  | H  |
      forces G
         | B  | C  | 1  |    | D4 | G2 | 7  | E  | H8 |
      leaving no space for A
     3b. |    | C  | 1  | B  | D  |    | 7  | E4 |    |
      forces the first square to be G
         | G  | C  | 1  | B8 | D  |    | 7  | E4 |    |
     forces H
         | G  | C  | 1  | B8 | D  |  2 | 7  | E4 | H  |
     forces I
         | G  | C5 | 1  | B8 | D  | I2 | 7  | E4 | H  |
     yielding the solution
         | G9 | C5 | 1F | B8 | D3 | I2 | A7 | E4 | H6 |
    

Mail to Ken